Download as docx, pdf, or txt
Download as docx, pdf, or txt
You are on page 1of 12

FNP Predictor Study 64 Complete Questions and Answers graded A+>.

A 32 year old man has acute bronchitis. He is a non smoker and is otherwise healthy.
How should he be managed?
A. Treat symptoms only
B. Prescribe a macrolide antibiotic.
C. Prescribe an an antitussive.
D. Prescribe an antibiotic and antitussive. - Correct answerA. Treat symptoms only

An 84 year old male has acute bronchitis. Which of the following tests would you
perform as the Nurse Practitioner?
A. Sputum culture
B. Chest X-ray
C. Chest CT
D. Chest MRI - Correct answerB. Chest X-ray

A 58 year old male has acute bronchitis. He is a heavy smoker, but is otherwise healthy.
What is the most likely etiology?
A. Viral infection
B. S. pneumoniae
C. B. Pertussis
D. Haemophilus influenzae - Correct answerD. Haemophilus influenzae

A 76 year old man has acute productive cough, fever 100-101 F for the past 3-4 days.
He is a non smoker and is otherwise healthy. Which likely diagnosis should be included
in your differential?
A. Acute bronchitis
B. Pneumonitis
C. Pneumonia
D. COPD
E. Bronchiectasis
F. Pertussis - Correct answerC. Pneumonia

Which of the following warrant a chest x-ray when an acute cough is present?
A. Temp 100.4
B. HR 86
C. Rales, consolidation
D. >65 years of age
E. RR 26 - Correct answerA. C. E.
Abnormal vital signs (increased RR or HR, temp >100.4)
Rales, consolidation
>75 years of age with cough
A 76 year old male has a chest x-ray that shows infiltrates. Which of the following
medication is first-line treatment?
A. Amoxicillin 1 gram TID
B. Doxycycline 100 mg BID
C. Azithromycin 500 mg on first day, then 250 mg daily
D. Clarithromycin 500 mg BID - Correct answerA. Amoxicillin 1 gram TID

A 76 year old male with a he of COPD, diabetes mellitus, and alcoholism has a chest x-
ray that shows infiltrates. Which of the following medications is first line treatment?
A. Amoxicillin
B. Levofloxacin
C. Azithromycin
D. Doxycycline - Correct answerB. Levofloxacin

A 76 year old male with a he of COPD, diabetes mellitus, and alcoholism has a chest x-
ray that shows infiltrates. Which of the following is the most likely pathogen?
A. H. flu
B. S. Pneumoniae
C. B. Pertussis
D. Viral infection - Correct answerA. H. flu.

A 56 year old male has suspected COPD. Which of the following symptoms are
indicative of COPD?
A. Progressive dyspnea
B. Acute cough
C. Sputum productive over the past 2 months
D. FEV1/FVC ratio >0.70
E. Alpha-1 antitrypsin deficiency - Correct answerA. C. E.
Chronic cough >8 weeks and FEV1/FVC ration <0.70.

A 56 year old male has been diagnosed with COPD. Which of the follow medications is
considered first-line?
A. SABAs (albuterol)
B. LABAs (salmeterol)
C. Inhaled anticholinergics (ipratropium)
D. LAAC (aclindinium) - Correct answerA. SABAs (albuterol) is first-line

Which of the following diagnoses decrease Hgb over time?


A. COPD Stage 3
B. DM with A1C 13.8
C. Testosterone use
D. Resident of Denver, CO
E. 84 years old - Correct answerA. B. E.
A 76 year old male presents with pinpoint non-blanching spots that measure less than 2
mm in size on his bilateral feet. Which of the following labs would be ordered?
A. Serum iron levels
B. B12 and folate levels
C. PT/INR
D. CBC - Correct answerD. CBC for petechiae/thrombocytopenia

A 14 year old female presents with a CBC that shows close poly and lymphocytes.
Which is the most likely etiology?
A. H. Flu.
B. Leukemia
C. Viral infection
D. B. Pertussis - Correct answerC. Viral infection

Patient has a CBC with the following values. What is the likely diagnosis?
WBC 2.9
Hgb 11.0
Hct 33.0
Poly% 81
Lymphs% 2
Mono%
Eos% 0.1
Bands 1

A. G6PD deficiency
B. Lead toxicity
C. Leukemia
D. B12, folate deficiency - Correct answerC. Leukemia

A 46 year old female presents with an A1C of 5.8%. Which of the following is the initial
management?
A. Biguanide
B. Lifestyle modifications
C. Repeat A1C in 3 months.
D. SGLT-2 inhibitor - Correct answerB. Lifestyle modifications

A 25 year old woman with newly diagnosed IFG, A1C 5.9% and is pregnant. Which
medication would be a first choice to prescribe?
A. Metformin
B. Glipizide
C. Liraglutide (Victoria)
D. Sitagliptin (Januvia) - Correct answerA. Metformin

A 56 year old female with T2DM and obesity was diagnosed 1 year ago. She takes
Metformin and had an MI 2 weeks ago. Her A1C is 7.5% today. Which medication class
would be considered for her?
A. S-GLT2
B. sulfonylurea
C. Meglitinide
D. DPP-4 inhibitor - Correct answerA. S-GLT 2, such as empagliflozin (Jardiance)

Which of the following doses is appropriate when starting basal insulin? Select all that
apply.
A. 0.1-0.2 u/kg
B. 2-4 u/kg
C. 10 units
D. 15 units - Correct answerA. C.

A 43 year old female has been diagnosed with hypothyroidism. Which of the following
symptoms would you expect her to be experiencing? Select all that apply.
A. Fatigue
B. Heat intolerance
C. Muscle weakness
D. Muscle cramps
E. Depression
F. Fine, silky hair
G. Skin dryness - Correct answerA. D. E. G.

Which metabolic abnormality is associated with hypothyroidism?


A. increased LDL values
B. Hyponatremia
C. Increased MCV
D. Elevated CK - Correct answerA. Increased LDL values

A 30 year old woman with a BMI of 31 has symptoms of hypothyroidism. Her TSH is
35.6 mU/L. When calculating replacement of levothyroxine, what must be determined
how much replacement is needed?
A. Actual weight
B. Ideal body weight
C. BMI
D. TSH - Correct answerB. Ideal body weight

A 45 year old patient has subclinical hypothyroidism, TSH 6.2. What are the major risks
of prescribing levothyroxine?
A. There are very few risks
B. Hyperlipidemia, atrial fibrillation
C. Accelerated bone loss, atrial fibrillation
D. She may develop hypertension and tachycardia - Correct answerC. Accelerated
bone loss, atrial fibrillation
A 58 year old female is found to be euthyroid. Which of the following is the next step of
care?
A. Repeat TSH in 4-6 weeks.
B. Increase levothyroxine until therapeutic.
C. Monitor TSH annually unless symptoms develop
D. Decrease levothyroxine until therapeutic. - Correct answerC. Monitor TSH
annually unless symptoms develop.

A 44 year old obese woman complains of abdominal pain. What physical assessment
finding might indicate that the etiology of her pain is acute cholecystitis?
A. Pain at McBurney's point.
B. Positive Murthy's sign
C. Severe epigastric pain
D. Thinking bowel sounds - Correct answerB. Positive Murphy's sign

A 68 year old male has LLQ abdominal pain and suspected diverticulitis. Which initial
diagnostic test would be ordered?
A. Ultrasound
B. Abdominal CT
C. Abdominal MRI
D. KUB X-ray - Correct answerA. Ultrasound

A 34 year old woman states that she has had diarrhea for 1 week. What factor should
make the provider strongly consider inflammatory bowel disease?
A. CBC demonstrates elevated WBCs.
B. She has a family history of irritable bowel syndrome.
C. She feels tired, weak, and has a fever.
D. She is having bloody nocturnal diarrhea. - Correct answerD. She is having
bloody nocturnal diarrhea.

What is the frequency of colonoscopy screening for adults? What is the frequency if the
patient is found to have polyps?
A. annually
B. 1-3 years
C. 5 years
D. 10 years - Correct answerD. And C.

A 68 year old male has the following lab values. He states that he feels well. What
should be done next?
WBC = 10
ALT = 348
AST = 148

A. repeat enzymes in 2 weeks


B. Order alkaline phosphatase
C. Order a hepatitis panel
D. Repeat the labs now - Correct answerC. Order a hepatitis panel

Each of the following patients have an elevated alkaline phosphatase. Which two of the
following choices indicate pathology?
A. Adolescent
B. Pregnant patient
C. 68 year old man with hematochezia
D. 56 year old man with prostate cancer - Correct answerC and D

A patient presents with these lab values. Which of the follow conclusion is correct?
(-) HBsAg
(-) anti-HBs
(-) anti-HBc

A. No current or past infection or immunity.


B. Immunity from immunization.
C. Chronic Hepatitis B
D. Immunity secondary past infection - Correct answerA. No current or past
infection or immunity.

A patient presents with these lab values. Which of the follow conclusion is correct?
(-) HBsAg
(+) anti-HBs
(-) anti-HBc

A. No current or past infection or immunity.


B. Immunity from immunization.
C. Chronic Hepatitis B
D. Immunity secondary past infection - Correct answerB. Immunity fromm
immunization

A patient presents with these lab values. Which of the follow conclusion is correct?
(+) HBsAg
(-) IgM anti-HBc
(-) anti-HBs
(+) anti-HBc

A. No current or past infection or immunity.


B. Immunity from immunization.
C. Chronic Hepatitis B
D. Immunity secondary past infection - Correct answerC. Chronic Hepatitis B

A patient presents with these lab values. Which of the follow conclusion is correct?
(-) HBsAg
(+) anti-HBs
(+) anti-HBc
A. No current or past infection or immunity.
B. Immunity from immunization.
C. Chronic Hepatitis B
D. Immunity secondary past infection - Correct answerD. Immunity secondary to
past infection

A patient presents with these lab values. Which of the follow conclusion is correct?
(+) HBsAg
(-) anti-HBs
(+) IgM anti-HBc
(+) anti-HBc

A. No current or past infection or immunity.


B. Immunity from immunization.
C. Chronic Hepatitis B
D. Immunity secondary past infection
E. Acute Hepatitis B - Correct answerE. Acute Hepatitis B

Which of the following at contraindications for combination hormonal contraceptives?


Select all that apply.
A. Dysmenorrhea
B. Migraines with aura
C. Hx of ischemic stroke
D. Hyperprolactinemia - Correct answerB and C

A 27 year old female has been diagnosed with bacterial vaginitis. Which choices listed
below are possible risk factors?
A. New sexual partner
B. Frequent condom use
C. Douching
D. Cooper IUD - Correct answerA. C. D.

A patient presents with malodorous vaginal discharge. Which of the following should be
included in your differential diagnoses? Select all that apply.
A. Vulvovaginal candidiasis
B. Bacterial vaginosis
C. HIV
D. Trichomoniasis - Correct answerB and D

A 68 year old female presents with vaginal itching and burning with dyspareunia. Which
of the following is treatment?
A. Estrogen PV
B. Symptom management
C. Fluconazole orally
D. Miconazole vaginally - Correct answerA. Estrogen PV

A 65 year old female has her first DEXA scan for screening for osteoporosis. Which of
the following T scores indicate osteoporosis? Indicate osteopenia?
A. >2.5
B. 1.0 to 2.5
C. -1.0 to -2.5
D. <-2.5 - Correct answerD. And C.

A 73 year old female is found to have a T score of <-2.5. Which of the following is
management for this score?
A. Weight bearing exercises.
B. Smoking cessation
C. Daily corticosteroid use
D. Calcium 1200 mg daily - Correct answerA. B. D.

Which medications could be safely initiated during an acute attack of gout?


A. allopurinol
B. Naproxen
C. Indomethacin
D. Prednisone - Correct answerB, C, D

A 27 year old pregnancy female presents with left thumb pain. Which test or symptoms
would be positive for DeQuervain's tenosynovitis?
A. Finkelstein
B. Snuffbox tenderness
C. Pain between 10 and 60 degrees of the Straight Leg Raise
D. Painful upon wakening - Correct answerA. Finkelstein

A 48 year old male presents after falling. He reports hanging on an outstretched hand,
Which test or symptoms would be positive for Scaphoid Fracture of the affected hand?
A. Finkelstein
B. Snuffbox tenderness
C. Pain between 10 and 60 degrees of the Straight Leg Raise
D. Painful upon wakening - Correct answerB. Snuffbox tenderness

A 68 year old female presents with a sharp, burning pain that radiates down her leg to
her foot. Which test or symptoms would be positive for sciatica?
A. Finkelstein
B. Snuffbox tenderness
C. Pain between 10 and 60 degrees of the Straight Leg Raise
D. Painful upon wakening - Correct answerC. Pain between 10 and 60 degrees of
the Straight Leg Raise
A 16 year old male is found to have Osgood Schlager disease. Which of the following
findings are characteristic of this? Select all that apply.
A. Erythema, warmth of the tibial tuberosity
B. Pain reproduced by extending the knee against resistance.
C. Pain occurs over the patellar tendon.
D. Straight leg raise is painless. - Correct answerB. And D.

A 56 year old male complains of pain in his lower legs when he walks. He gets complete
relief when he stops walking and rests. What is the NP likely to elicit from his medical
history?
A. Diabetes
B. Smoking
C. Dyslipidemia
D. Regular exercise - Correct answerA, B, C

A 62 year old male presents with angina after a syncopal episode. He is also suspected
to have congestive heart failure. You auscultate S4 with a narrow pulse pressure that is
loudest at the upper sternal border. Which of the following murmurs is this?
A. Aortic Stenosis
B. Mitral Regurgitation
C. Mitral Valve Prolapse
D. Mitral Stenosis - Correct answerA. Aortic Stenosis

A 58 year old female with a hx of complications due to syphilis presents with decreased
exercise tolerance, chest pain, and palpitations. Her HR is 110. You auscultate a
diastolic murmur. Which of the following is this murmur?
A. Aortic Stenosis
B. Atrial Regurgitation
C. Mitral Valve Prolapse
D. Mitral Stenosis - Correct answerB. Atrial Regurgitation

A 24 year old female has rheumatic fever with a new onset of atrial fibrillation and
exercise intolerance. Which murmur would you expect to be heard?
A. Aortic Stenosis
B. Mitral Regurgitation
C. Mitral Valve Prolapse
D. Mitral Stenosis - Correct answerD. Mitral Stenosis

A 53 year old male with a reported unknown congenital heart defect presents today with
symptoms of fatigue. An ECG shows atrial fibrillation. Which murmur would you expect
to auscultate?
A. Aortic Stenosis
B. Mitral Regurgitation
C. Mitral Valve Prolapse
D. Mitral Stenosis - Correct answerB. Mitral Regurgitation
A 16 year old female is found to have redundancy of the mitral valve leaflets. You hear
a midsystolic click heard best the the apex and left sternal border. Which murmur is
this?
A. Aortic Stenosis
B. Mitral Regurgitation
C. Mitral Valve Prolapse
D. Mitral Stenosis - Correct answerC. Mitral Valve Prolapse

Which of the following organisms are Gram-Positive? Select all that apply.
A. Staphylococcus
B. Streptococcus
C. Enterococcus
D. E. Coli
E. C. difficile - Correct answerA B C E

Which of the following organisms are susceptible to penicillins?


A. Streptococcus
B. Staphylococcus
C. Strep pneumoniae
D. MRSA - Correct answerA. C.

A 32 year old pregnant female is diagnosed with Strep. Which of the following
medications is to be prescribed?
A. Penicillins (Amoxicillin)
B. 2nd generation fluoroquinolone (ciprofloxacin)
C. 3rd generation or "respiratory" fluoroquinolone (levofloxacin)
D. nitrofurantoin (Macrobid) - Correct answerC. 3rd generation or "respiratory"
fluoroquinolone

Which of the following medications has an indication only for E. Coli?


A. Penicillin
B. Ciprofloxacin (Cipro)'
C. Cephalexin (Keflex)
D. nitrofurantoin (Macrobid) - Correct answerD. Nitrofurantoin (Macrobid)

Which of the follow fluoroquinolone is used "below the belt" or for urinary symptoms?
A. ciprofloxacin (Cipro)
B. levofloxacin (Levaquin)
C. moxifloxacin (Avelox)
D. gemifloxacin (Factive) - Correct answerA. Ciprofloxacin (Levaquin)

Which of the follow fluoroquinolone is used "above the belt" or for repiratory symptoms?
A. ciprofloxacin (Cipro)
B. levofloxacin (Levaquin)
C. moxifloxacin (Avelox)
D. gemifloxacin (Factive) - Correct answerB. Levofloxacin (Levaquin)

Which of the following medications is considered to be first choice for an uncomplicated


UTI?
A. cephalosporins
B. Nitrofurantoin
C. Ciprofloxacin
D. Sulfamethoxazole/trimethoprim (Bactrim) - Correct answerB. Nitrofurantoin

A 26 year old female presented with burning, frequency, urgency. A UA has the
following values. Which of the following tests would be ordered next?
Blood (-)
Nitrites (-)
Leuko (+)

A. KUB x ray
B. Urine culture
C. STI testing
D. Pap smear - Correct answerC. STI testing

A 76 year old female has a urinary tract infection. Which is of the following would be
involved in assessment and treatment of this patient?
A. Nitrofurantoin (Macrobid) for 7-10 days
B. Sulfamethoxazole/trimethoprim (Bactrim) for 3 days
C. Inquire about antibiotic use over the past three months
D. Order a urine culture
E. Nitrofurantoin (Macrobid) for 3 days - Correct answerA C D

A 26 year old female who is not pregnant and otherwise healthy has a urinary tract
infection. Which is of the following would be involved in assessment and treatment of
this patient?
A. Nitrofurantoin (Macrobid) for 7-10 days
B. Sulfamethoxazole/trimethoprim (Bactrim) for 3 days
C. Inquire about antibiotic use over the past three months
D. Order a urine culture
E. Nitrofurantoin (Macrobid) for 3 days - Correct answerC. D. E.

A 56 year old male presents with suspected kidney stones. Which of the following tests
is the gold standard for diagnosis? Which medication is indicated if otherwise not
contraindicated?
A. MRI
B. Helical CT scan
C. Ultrasound
D. Toradol injection
E. Amoxicillin
F. 2nd generation fluoroquinolone (ciprofloxacin (Cipro)) - Correct answerB, D

A 68 year old female presents with hematuria, temp of 100.8, and CVA tenderness.
Which of the following medications is indicated at this time?
A. 2nd generation fluoroquinolone (ciprofloxacin (Cipro)) for 7 days
B. 1st generation cephalosporin (Cephalexin (Keflex)) for 10 days
C. Nitrofurantoin (Macrobid) for 3 days
D. 3rd generation cephalosporins extended release (ceftriaxone (Rocephine)) for 14
days - Correct answerA. 2nd generation fluoroquinolone (ciprofloxacin (Cipro)) for 7
days

Which of the following patients would not be screened for HIV?


A. Routine screening for 13-64 year old patients
B. Currently treated for TB
C. Diagnosed with syphilis, GC, or chlamydia
D. Pregnant women once per trimester - Correct answerD. Pregnant women once
per trimester

You might also like